Dubbi integrale multiplo

Messaggioda curioso54 » 21/02/2024, 00:21

Salve. Recentemente mi sono imbattuto sulla risoluzione degli integrali tripli e ho fondamentalmente notato che si può risalire agli estremi di integrazione di un dato insieme di integrazione per via puramente algebrica (ponendo bene le condizioni di esistenza di tuttw le funzioni che vengono fuori con i calcoli) senza il bisogno di rappresentare il dominio graficamente. Però ciò non mi torna nel caso del volume di un anello sferico usando le coordinate cilindriche (so quale è il risultato, ma io voglio ricavarlo utilizzando le coordinate cilindriche).
Ho impostato l'integrale triplo il cui integrando è 1, riscritto l'insieme di integrazione in modo da estrapolare gli estremi di integrazione e torna quasi tutto tranne un radice di 3 che viene fuori. Come è possibile?

(Domani scriverò tutti i passaggi così potete vedere dove sbaglio, ma se qualcuno impostasse già il tutto con le coordinate cilindriche affinché possa notare io stesso dove sbaglio mi farebbe un gran favore).

Ringrazio in anticipo
curioso54
Starting Member
Starting Member
 
Messaggio: 1 di 19
Iscritto il: 13/04/2023, 13:01

Re: Dubbi integrale multiplo

Messaggioda pilloeffe » 21/02/2024, 07:54

Ciao curioso54,

Benvenuto sul forum!

curioso54 ha scritto:Ho impostato l'integrale triplo il cui integrando è 1, riscritto l'insieme di integrazione in modo da estrapolare gli estremi di integrazione e torna quasi tutto tranne un radice di 3 che viene fuori. Come è possibile?

E' praticamente impossibile capire dove sbagli (se sbagli, magari è un errore di stampa nel risultato... :wink: Per inciso, ti sei ricordato dello jacobiano della trasformazione in coordinate cilindriche?) se non posti i passaggi o almeno il testo dell'integrale triplo col suo dominio $T$, che se ho ben capito è una cosa del genere:

$\int \int \int_T f(x, y, z) \text{d}x \text{d}y \text{d}z = \int \int \int_T 1 \text{d}x \text{d}y \text{d}z $

$ T = ? $

Codice:
$\int \int \int_T f(x, y, z) \text{d}x  \text{d}y  \text{d}z = \int \int \int_T 1 \text{d}x  \text{d}y  \text{d}z $

$ T = ? $


Se scrivi anche cosa dovrebbe risultare meglio ancora... :wink:
pilloeffe
Cannot live without
Cannot live without
 
Messaggio: 5838 di 10595
Iscritto il: 07/02/2017, 15:45
Località: La Maddalena - Modena

Re: Dubbi integrale multiplo

Messaggioda curioso54 » 21/02/2024, 13:01

Pensavo che il volume dell'anello sferico fosse ovvio, ma provvederò a scrivere il risultato non appena torno a casa
curioso54
Starting Member
Starting Member
 
Messaggio: 2 di 19
Iscritto il: 13/04/2023, 13:01

Re: Dubbi integrale multiplo

Messaggioda pilloeffe » 21/02/2024, 16:17

curioso54 ha scritto:Pensavo che il volume dell'anello sferico fosse ovvio

Mah, per me no, ma può anche essere che sia un problema mio... :wink:
curioso54 ha scritto:provvederò a scrivere il risultato non appena torno a casa

Il risultato va bene, ma ancora più importante del risultato è che tu scriva per bene com'è definito $T$
pilloeffe
Cannot live without
Cannot live without
 
Messaggio: 5840 di 10595
Iscritto il: 07/02/2017, 15:45
Località: La Maddalena - Modena

Re: Dubbi integrale multiplo

Messaggioda curioso54 » 23/02/2024, 00:28

$\int \int \int_T 1 \text{d}x\text{d}x\text{d}x $

$T = {(x,y,z)\in \RR^3 : x^2+y^2+z^2<1, x^2+y^2+z^2<4} $


$\phi(T)=\left\
begin{aligned}
r^2+z^2. &>1 \\

r^2+z^2. &<4
\end{aligned}
\right $











Ho fatto bene alle manipolazioni nell'insieme di integrazione ma non capisco come venga fuori un risultato del genere


Il volume dell'anello sferico è $\frac{4\pi}{3}(R_1^3-R_2^3)$ , e nel caso di raggi 2 e 1 deve tornare $\frac{28}{3}\pi$

Qualcuno può farmi capire dove ho sbagliato, attenendosi alle coordinate cilindriche??
Ultima modifica di curioso54 il 23/02/2024, 10:29, modificato 19 volte in totale.
curioso54
Starting Member
Starting Member
 
Messaggio: 3 di 19
Iscritto il: 13/04/2023, 13:01

Re: Dubbi integrale multiplo

Messaggioda curioso54 » 23/02/2024, 01:01

pilloeffe ha scritto:Ciao curioso54,

Benvenuto sul forum!

curioso54 ha scritto:Ho impostato l'integrale triplo il cui integrando è 1, riscritto l'insieme di integrazione in modo da estrapolare gli estremi di integrazione e torna quasi tutto tranne un radice di 3 che viene fuori. Come è possibile?

E' praticamente impossibile capire dove sbagli (se sbagli, magari è un errore di stampa nel risultato... :wink: Per inciso, ti sei ricordato dello jacobiano della trasformazione in coordinate cilindriche?) se non posti i passaggi o almeno il testo dell'integrale triplo col suo dominio $T$, che se ho ben capito è una cosa del genere:

$\int \int \int_T f(x, y, z) \text{d}x \text{d}y \text{d}z = \int \int \int_T 1 \text{d}x \text{d}y \text{d}z $

$ T = ? $

Codice:
$\int \int \int_T f(x, y, z) \text{d}x  \text{d}y  \text{d}z = \int \int \int_T 1 \text{d}x  \text{d}y  \text{d}z $

$ T = ? $


Se scrivi anche cosa dovrebbe risultare meglio ancora... :wink:


Mi sono ricordato dello jacobiano sì, ma non capisco dove sia il problema qui – algebricamente parlando – perché nelle parentesi graffe non vedo alcun errore di riscrittura del dominio: il raggio è per forza compreso tra 0 e 1 in quanto soddisfa le C.E di entrambe le funzioni $\sqrt(4-r^2)$ e $\sqrt(1-r^2)$
curioso54
Starting Member
Starting Member
 
Messaggio: 4 di 19
Iscritto il: 13/04/2023, 13:01

Re: Dubbi integrale multiplo

Messaggioda pilloeffe » 23/02/2024, 07:36

curioso54 ha scritto:Qualcuno può farmi capire dove ho sbagliato, attenendosi alle coordinate cilindriche??

A parte che è vietato postare immagini sul forum, in quanto i siti di hosting dopo poco tempo eliminano le immagini e quindi rendono il post ed il thread praticamente illeggibili, sicché ti inviterei a modificare il tuo post eliminando l'immagine e scrivendo semplicemente il dominio

$T = {(x,y, z) \in \RR^3 : x^2 + y^2 + z^2 > 1, x^2 + y^2 + z^2 < 4} $

Codice:
$T = {(x,y, z) \in \RR^3 : x^2 + y^2 + z^2 > 1, x^2 + y^2 + z^2 < 4} $

non capisco perché fai la somma di quei due integrali: per trovare il volume dell'anello io farei la differenza dei volumi delle due sfere, non la somma... :wink:
Comunque qui, trattandosi di sfere, è evidente che convengono di gran lunga le coordinate sferiche:

$T_s = {(\rho,\theta, \varphi) \in \RR^3 : 1 < \rho < 2, 0 \le \theta < 2\pi, 0 \le \varphi < \pi} $
pilloeffe
Cannot live without
Cannot live without
 
Messaggio: 5843 di 10595
Iscritto il: 07/02/2017, 15:45
Località: La Maddalena - Modena

Re: Dubbi integrale multiplo

Messaggioda curioso54 » 23/02/2024, 09:21

Non sapevo fosse vietato postare immagini, l'ho fatto perché non sono molto familiare con LaTex. Il mio obiettivo qui è di arrivare al risultato utilizzando le cilindriche e capire dove sbaglio nei procedimenti.
curioso54
Starting Member
Starting Member
 
Messaggio: 5 di 19
Iscritto il: 13/04/2023, 13:01

Re: Dubbi integrale multiplo

Messaggioda Mephlip » 23/02/2024, 09:56

@curioso54: Sbagli a risolvere le disequazioni. Osserva che, tenendo conto che \(r>0\), si ha:\[
[r^2+z^2<4] \implies [r^2 \le r^2+z^2<4] \implies [r^2<4] \implies [0 < r < 2]
\]E si ha:\[
[r^2+z^2<4] \implies [z^2<r^2+z^2<4] \implies [z^2<4] \iff [ \ |z| < 2 \ ]
\]Per determinare l'intervallo in cui varia \(z\), distinguiamo due casi su \(r\). Se \(0<r<1\), allora \(1-r^2>0\) e pertanto:\[
[1<r^2+z^2<4] \iff [1-r^2<z^2<4-r^2] \iff \left[\sqrt{1-r^2}<|z|<\sqrt{4-r^2}\right]
\]
Se invece \(1<r<2\), allora \(4-r^2>0\) e \(1-r^2<0\). Da quest'ultima, segue che \(z^2>1-r^2\) è vera per ogni \(z \in \mathbb{R}\) e quindi in questo caso si ha:\[
\left[1-r^2<z^2<4-r^2\right] \iff \left[z^2<4-r^2\right] \iff \left[ \ |z| < \sqrt{4-r^2} \right]
\]Ora, osserva che la funzione integranda e \(T\) sono pari in \(z\) (ossia, scambiando \(z\) con \(-z\) rimangono invariati sia funzione integranda sia insieme di integrazione), quindi l'integrale in esame è pari al doppio dell'integrale calcolato aggiungendo a \(T\) la condizione \(z>0\). Ma, con tale condizione aggiuntiva, è \(|z|=z\) e perciò le limitazioni su \(z\) sono \(\sqrt{1-r^2}<z<\sqrt{4-r^2}\) ed \(0<r<1\) o \(0<z<\sqrt{4-r^2}\) ed \(1<r<2\) .

Perciò, si ha:\[
\iiint_T \text{d}x\text{d}y\text{d}z=2 \cdot 2\pi \left[\int_0^1 \left(\int_{\sqrt{1-r^2}}^{\sqrt{4-r^2}}r\text{d}z\right)\text{d}r+\int_1^2 \left(\int_0^{\sqrt{4-r^2}}r\text{d}z\right)\text{d}r\right]=\frac{28}{3}\pi
\]
In sostanza, il tuo errore è stato quello di trascurare delle condizioni aggiuntive su \(r\): hai dato per scontato che le condizioni di realtà delle radici fossero gli unici vincoli sull'intervallo in cui varia \(r\), ma non è così (come puoi vedere nel passaggio "\(r^2<r^2+z^2<4\) implica \(0<r<2\)"). Per alcuni valori di \(0<r<2\), le limitazioni su \(z\) cambiano in accordo a quello che ho scritto su. In particolare, per alcuni valori ammissibili di \(r\) hai addirittura che \(1-r^2<0\) e quindi non puoi assolutamente estrarne la radice in questo contesto di analisi reale.

Se proprio non vuoi usare la parità su \(z\), basta ragionare similmente ma trovandosi in quell'inferno di casi che escono fuori dovendo considerare tutte le casistiche date dal valore assoluto su \(z\). Francamente, è un approccio decisamente masochista :-D.
A spoon can be used for more than just drinking soup. You can use it to dig through the prison you're locked in, or as a weapon to gouge the witch's eyes out. Of course, you can also use the spoon to continually sip the watery soup inside your eternal prison.
Avatar utente
Mephlip
Moderatore globale
Moderatore globale
 
Messaggio: 2355 di 3662
Iscritto il: 03/06/2018, 23:53

Re: Dubbi integrale multiplo

Messaggioda pilloeffe » 23/02/2024, 10:01

curioso54 ha scritto:Non sapevo fosse vietato postare immagini, l'ho fatto perché non sono molto familiare con LaTex.

Lo immaginavo, per questo ti ho scritto il codice per scrivere l'insieme $T$... :wink:
curioso54 ha scritto:Il mio obiettivo qui è di arrivare al risultato utilizzando le cilindriche e capire dove sbaglio nei procedimenti.

Hai compreso bene quanto
pilloeffe ha scritto:non capisco perché fai la somma di quei due integrali: per trovare il volume dell'anello io farei la differenza dei volumi delle due sfere, non la somma

Generalizzando ed insistendo ad usare le coordinate cilindriche, cosa che fra l'altro io non farei perché è del tutto evidente che qui siano molto più convenienti le coordinate sferiche, farei così:

$V_{\text{anello}}(R_1, R_2) = \int_0^{2\pi} \text{d}\theta \int_0^{R_1}[\int_{-\sqrt{R_1^2 - \rho^2 }}^{\sqrt{R_1^2 - \rho^2}} \text{d}z] \rho \text{d}\rho - \int_0^{2\pi} \text{d}\theta \int_0^{R_2}[\int_{-\sqrt{R_2^2 - \rho^2}}^{\sqrt{R_2^2 - \rho^2}} \text{d}z] \rho \text{d}\rho = $

$ = (4\pi)/3 R_1^3 - (4\pi)/3 R_2^3 = (4\pi)/3 (R_1^3 - R_2^3) $

Ovviamente nel caso particolare $R_1 = 2 $ e $R_2 = 1 $ si ha:

$V_{\text{anello}}(2, 1) = (4\pi)/3 (2^3 - 1^3) = (4\pi)/3 (8 - 1) = (28\pi)/3 $

Invece con le coordinate sferiche:

$V_{\text{anello}}(R_1, R_2) = \int_0^{2\pi} \text{d}\theta \int_0^{\pi} sin\varphi \text{d}\varphi \int_{R_2}^{R_1} \rho^2 \text{d}\rho = (4\pi)/3 (R_1^3 - R_2^3) $
pilloeffe
Cannot live without
Cannot live without
 
Messaggio: 5844 di 10595
Iscritto il: 07/02/2017, 15:45
Località: La Maddalena - Modena

Prossimo

Torna a Analisi matematica di base

Chi c’è in linea

Visitano il forum: Quasar3.14 e 1 ospite